Line Integral Help: Evaluating F ds on a Curve in 1st Quadrant

Click For Summary
The discussion centers on evaluating the line integral of the vector field F = (6(x^2)(y^2), 4(x^3)(y) + 5y^4) along the boundary of the first quadrant below the curve y = 1 - x^2. Participants confirm the use of Green's Theorem, noting that the vector field is conservative, leading to a line integral value of zero over the closed curve. This conclusion is supported by the condition that the partial derivatives dF1/dy and dF2/dx are equal. The consensus is that the integral evaluates to zero due to the conservative nature of the vector field. Overall, the evaluation aligns with the principles of vector calculus and Green's Theorem.
Kuma
Messages
129
Reaction score
0

Homework Statement



Trying to evaluate the following line integral:

integral F ds where F = (6(x^2)(y^2), 4(x^3)(y) + 5y^4)
and the path is the boundary curve of the first quadrant below y = 1-x^2 in a clockwise direction.

Homework Equations





The Attempt at a Solution



So since the curve is piecewise smooth closed simple and closed curve I can use greens theorem. Simply put I get an answer as 0 since dF1/dy = dF2/dx. Is that right?
 
Physics news on Phys.org
Kuma said:

Homework Statement



Trying to evaluate the following line integral:

integral F ds where F = (6(x^2)(y^2), 4(x^3)(y) + 5y^4)
and the path is the boundary curve of the first quadrant below y = 1-x^2 in a clockwise direction.

Homework Equations





The Attempt at a Solution



So since the curve is piecewise smooth closed simple and closed curve I can use greens theorem. Simply put I get an answer as 0 since dF1/dy = dF2/dx. Is that right?

I would agree with that.
 
Your vector field is conservative. So the line integral would be zero over any closed curve (even non-simple closed curve!). A vector field is conservative if it is the gradient of a potential function.
 
Question: A clock's minute hand has length 4 and its hour hand has length 3. What is the distance between the tips at the moment when it is increasing most rapidly?(Putnam Exam Question) Answer: Making assumption that both the hands moves at constant angular velocities, the answer is ## \sqrt{7} .## But don't you think this assumption is somewhat doubtful and wrong?

Similar threads

  • · Replies 4 ·
Replies
4
Views
2K
Replies
12
Views
2K
  • · Replies 10 ·
Replies
10
Views
2K
Replies
3
Views
2K
Replies
12
Views
4K
  • · Replies 2 ·
Replies
2
Views
1K
Replies
6
Views
3K
  • · Replies 1 ·
Replies
1
Views
2K
  • · Replies 2 ·
Replies
2
Views
1K
  • · Replies 6 ·
Replies
6
Views
1K